Proof that this product is transcendental (or not) [on hold]












0














The following products can be expressed using the q-Pochhammer symbol:



$Pi^{infty}_{n=0}left(1+e^{-n}right)=left(-1; frac{1}{e}right)_infty$



$Pi^{infty}_{n=1}left(1-e^{-n}right)=left(frac{1}{e}; frac{1}{e}right)_infty$



Must these two constants be transcendental? I believe they should, but I'm looking for how to prove if they are indeed or are not.










share|cite|improve this question















put on hold as off-topic by Math1000, jgon, Cesareo, KReiser, Holo yesterday


This question appears to be off-topic. The users who voted to close gave this specific reason:


  • "This question is missing context or other details: Please provide additional context, which ideally explains why the question is relevant to you and our community. Some forms of context include: background and motivation, relevant definitions, source, possible strategies, your current progress, why the question is interesting or important, etc." – Math1000, jgon, Cesareo, KReiser, Holo

If this question can be reworded to fit the rules in the help center, please edit the question.













  • Your thoughts? $,$
    – Math1000
    yesterday
















0














The following products can be expressed using the q-Pochhammer symbol:



$Pi^{infty}_{n=0}left(1+e^{-n}right)=left(-1; frac{1}{e}right)_infty$



$Pi^{infty}_{n=1}left(1-e^{-n}right)=left(frac{1}{e}; frac{1}{e}right)_infty$



Must these two constants be transcendental? I believe they should, but I'm looking for how to prove if they are indeed or are not.










share|cite|improve this question















put on hold as off-topic by Math1000, jgon, Cesareo, KReiser, Holo yesterday


This question appears to be off-topic. The users who voted to close gave this specific reason:


  • "This question is missing context or other details: Please provide additional context, which ideally explains why the question is relevant to you and our community. Some forms of context include: background and motivation, relevant definitions, source, possible strategies, your current progress, why the question is interesting or important, etc." – Math1000, jgon, Cesareo, KReiser, Holo

If this question can be reworded to fit the rules in the help center, please edit the question.













  • Your thoughts? $,$
    – Math1000
    yesterday














0












0








0







The following products can be expressed using the q-Pochhammer symbol:



$Pi^{infty}_{n=0}left(1+e^{-n}right)=left(-1; frac{1}{e}right)_infty$



$Pi^{infty}_{n=1}left(1-e^{-n}right)=left(frac{1}{e}; frac{1}{e}right)_infty$



Must these two constants be transcendental? I believe they should, but I'm looking for how to prove if they are indeed or are not.










share|cite|improve this question















The following products can be expressed using the q-Pochhammer symbol:



$Pi^{infty}_{n=0}left(1+e^{-n}right)=left(-1; frac{1}{e}right)_infty$



$Pi^{infty}_{n=1}left(1-e^{-n}right)=left(frac{1}{e}; frac{1}{e}right)_infty$



Must these two constants be transcendental? I believe they should, but I'm looking for how to prove if they are indeed or are not.







infinite-product






share|cite|improve this question















share|cite|improve this question













share|cite|improve this question




share|cite|improve this question








edited yesterday

























asked yesterday









El Ectric

1549




1549




put on hold as off-topic by Math1000, jgon, Cesareo, KReiser, Holo yesterday


This question appears to be off-topic. The users who voted to close gave this specific reason:


  • "This question is missing context or other details: Please provide additional context, which ideally explains why the question is relevant to you and our community. Some forms of context include: background and motivation, relevant definitions, source, possible strategies, your current progress, why the question is interesting or important, etc." – Math1000, jgon, Cesareo, KReiser, Holo

If this question can be reworded to fit the rules in the help center, please edit the question.




put on hold as off-topic by Math1000, jgon, Cesareo, KReiser, Holo yesterday


This question appears to be off-topic. The users who voted to close gave this specific reason:


  • "This question is missing context or other details: Please provide additional context, which ideally explains why the question is relevant to you and our community. Some forms of context include: background and motivation, relevant definitions, source, possible strategies, your current progress, why the question is interesting or important, etc." – Math1000, jgon, Cesareo, KReiser, Holo

If this question can be reworded to fit the rules in the help center, please edit the question.












  • Your thoughts? $,$
    – Math1000
    yesterday


















  • Your thoughts? $,$
    – Math1000
    yesterday
















Your thoughts? $,$
– Math1000
yesterday




Your thoughts? $,$
– Math1000
yesterday










0






active

oldest

votes

















0






active

oldest

votes








0






active

oldest

votes









active

oldest

votes






active

oldest

votes

Popular posts from this blog

Mario Kart Wii

What does “Dominus providebit” mean?

Antonio Litta Visconti Arese